Finding the sum of a specific alternating series












1












$begingroup$


The series $$sum_{n=1}^infty dfrac{(-1)^{n +1} (n+1)}{n!}$$ can be easily proven by using Leibniz's test to be convergent. But I am finding a problem in finding it's sum.
Some help is much required. Thank you










share|cite|improve this question











$endgroup$












  • $begingroup$
    Try breaking it into two sums.
    $endgroup$
    – saulspatz
    Jan 16 at 15:10
















1












$begingroup$


The series $$sum_{n=1}^infty dfrac{(-1)^{n +1} (n+1)}{n!}$$ can be easily proven by using Leibniz's test to be convergent. But I am finding a problem in finding it's sum.
Some help is much required. Thank you










share|cite|improve this question











$endgroup$












  • $begingroup$
    Try breaking it into two sums.
    $endgroup$
    – saulspatz
    Jan 16 at 15:10














1












1








1





$begingroup$


The series $$sum_{n=1}^infty dfrac{(-1)^{n +1} (n+1)}{n!}$$ can be easily proven by using Leibniz's test to be convergent. But I am finding a problem in finding it's sum.
Some help is much required. Thank you










share|cite|improve this question











$endgroup$




The series $$sum_{n=1}^infty dfrac{(-1)^{n +1} (n+1)}{n!}$$ can be easily proven by using Leibniz's test to be convergent. But I am finding a problem in finding it's sum.
Some help is much required. Thank you







real-analysis calculus






share|cite|improve this question















share|cite|improve this question













share|cite|improve this question




share|cite|improve this question








edited Jan 16 at 15:09







dame j

















asked Jan 16 at 15:08









dame jdame j

19310




19310












  • $begingroup$
    Try breaking it into two sums.
    $endgroup$
    – saulspatz
    Jan 16 at 15:10


















  • $begingroup$
    Try breaking it into two sums.
    $endgroup$
    – saulspatz
    Jan 16 at 15:10
















$begingroup$
Try breaking it into two sums.
$endgroup$
– saulspatz
Jan 16 at 15:10




$begingroup$
Try breaking it into two sums.
$endgroup$
– saulspatz
Jan 16 at 15:10










4 Answers
4






active

oldest

votes


















3












$begingroup$

$$(-1)^{n+1}cdotdfrac{n+1}{n!}=dfrac{(-1)^{n-1}}{(n-1)!}-dfrac{(-1)^n}{n!}=f(n-1)-f(n)$$



where $f(m)=dfrac{(-1)^m}{m!}$



See https://en.m.wikipedia.org/wiki/Telescoping_series






share|cite|improve this answer









$endgroup$





















    2












    $begingroup$

    Note that, since all of the relevant sums converge absolutely



    $$sumlimits_{i=1}^infty frac{(-1)^{n+1}(n+1)}{n!} = sumlimits_{i=1}^inftyleft(frac{(-1)^{n+1}}{(n-1)!} + frac{(-1)^{n+1}}{n!}right).$$



    Notice that (again, since everything in sight converges absolutely) the second half of the $n$th term cancels with the first half of the $(n+1)$st term, so our sum telescopes to



    $$frac{(-1)^2}{0!} = 1.$$






    share|cite|improve this answer









    $endgroup$





















      0












      $begingroup$

      $
      sum_{n=1}^{infty}frac{(-1)^{n+1}(n+1)}{n!}\
      =sum_{n=0}^{infty}frac{(-1)^{n}(n+2)}{(n+1)!}\
      =sum_{n=0}^{infty}frac{(-1)^{n}}{n!}+sum_{n=0}^{infty}frac{(-1)^{n}}{(n+1)!}\
      =sum_{n=0}^{infty}frac{(-1)^{n}}{n!}+sum_{n=1}^{infty}frac{(-1)^{n+1}}{n!}\
      =sum_{n=0}^{infty}frac{(-1)^{n}}{n!}-sum_{n=1}^{infty}frac{(-1)^n}{n!}\
      =1
      $






      share|cite|improve this answer









      $endgroup$





















        0












        $begingroup$

        Starting with
        $$e^{-x} = sum_{n=0}^{infty} (-1)^n frac{1}{n!} x^n$$
        Multiply by $x$
        $$x e^{-x} = sum_{n=0}^{infty} (-1)^n frac{1}{n!} x^{n+1}$$
        Differentiate with respect to $x$
        $$e^{-x}-x e^{-x} = sum_{n=0}^{infty} (-1)^n frac{n+1}{n!} x^n = 1 - sum_{n=1}^{infty} (-1)^{n+1} frac{n+1}{n!} x^n$$
        Now set $x=1$, with result
        $$0 = 1 - sum_{n=1}^{infty} (-1)^{n+1} frac{n+1}{n!} x^n$$
        so the sum is $1$.






        share|cite|improve this answer









        $endgroup$














          Your Answer





          StackExchange.ifUsing("editor", function () {
          return StackExchange.using("mathjaxEditing", function () {
          StackExchange.MarkdownEditor.creationCallbacks.add(function (editor, postfix) {
          StackExchange.mathjaxEditing.prepareWmdForMathJax(editor, postfix, [["$", "$"], ["\\(","\\)"]]);
          });
          });
          }, "mathjax-editing");

          StackExchange.ready(function() {
          var channelOptions = {
          tags: "".split(" "),
          id: "69"
          };
          initTagRenderer("".split(" "), "".split(" "), channelOptions);

          StackExchange.using("externalEditor", function() {
          // Have to fire editor after snippets, if snippets enabled
          if (StackExchange.settings.snippets.snippetsEnabled) {
          StackExchange.using("snippets", function() {
          createEditor();
          });
          }
          else {
          createEditor();
          }
          });

          function createEditor() {
          StackExchange.prepareEditor({
          heartbeatType: 'answer',
          autoActivateHeartbeat: false,
          convertImagesToLinks: true,
          noModals: true,
          showLowRepImageUploadWarning: true,
          reputationToPostImages: 10,
          bindNavPrevention: true,
          postfix: "",
          imageUploader: {
          brandingHtml: "Powered by u003ca class="icon-imgur-white" href="https://imgur.com/"u003eu003c/au003e",
          contentPolicyHtml: "User contributions licensed under u003ca href="https://creativecommons.org/licenses/by-sa/3.0/"u003ecc by-sa 3.0 with attribution requiredu003c/au003e u003ca href="https://stackoverflow.com/legal/content-policy"u003e(content policy)u003c/au003e",
          allowUrls: true
          },
          noCode: true, onDemand: true,
          discardSelector: ".discard-answer"
          ,immediatelyShowMarkdownHelp:true
          });


          }
          });














          draft saved

          draft discarded


















          StackExchange.ready(
          function () {
          StackExchange.openid.initPostLogin('.new-post-login', 'https%3a%2f%2fmath.stackexchange.com%2fquestions%2f3075841%2ffinding-the-sum-of-a-specific-alternating-series%23new-answer', 'question_page');
          }
          );

          Post as a guest















          Required, but never shown

























          4 Answers
          4






          active

          oldest

          votes








          4 Answers
          4






          active

          oldest

          votes









          active

          oldest

          votes






          active

          oldest

          votes









          3












          $begingroup$

          $$(-1)^{n+1}cdotdfrac{n+1}{n!}=dfrac{(-1)^{n-1}}{(n-1)!}-dfrac{(-1)^n}{n!}=f(n-1)-f(n)$$



          where $f(m)=dfrac{(-1)^m}{m!}$



          See https://en.m.wikipedia.org/wiki/Telescoping_series






          share|cite|improve this answer









          $endgroup$


















            3












            $begingroup$

            $$(-1)^{n+1}cdotdfrac{n+1}{n!}=dfrac{(-1)^{n-1}}{(n-1)!}-dfrac{(-1)^n}{n!}=f(n-1)-f(n)$$



            where $f(m)=dfrac{(-1)^m}{m!}$



            See https://en.m.wikipedia.org/wiki/Telescoping_series






            share|cite|improve this answer









            $endgroup$
















              3












              3








              3





              $begingroup$

              $$(-1)^{n+1}cdotdfrac{n+1}{n!}=dfrac{(-1)^{n-1}}{(n-1)!}-dfrac{(-1)^n}{n!}=f(n-1)-f(n)$$



              where $f(m)=dfrac{(-1)^m}{m!}$



              See https://en.m.wikipedia.org/wiki/Telescoping_series






              share|cite|improve this answer









              $endgroup$



              $$(-1)^{n+1}cdotdfrac{n+1}{n!}=dfrac{(-1)^{n-1}}{(n-1)!}-dfrac{(-1)^n}{n!}=f(n-1)-f(n)$$



              where $f(m)=dfrac{(-1)^m}{m!}$



              See https://en.m.wikipedia.org/wiki/Telescoping_series







              share|cite|improve this answer












              share|cite|improve this answer



              share|cite|improve this answer










              answered Jan 16 at 15:12









              lab bhattacharjeelab bhattacharjee

              228k15158279




              228k15158279























                  2












                  $begingroup$

                  Note that, since all of the relevant sums converge absolutely



                  $$sumlimits_{i=1}^infty frac{(-1)^{n+1}(n+1)}{n!} = sumlimits_{i=1}^inftyleft(frac{(-1)^{n+1}}{(n-1)!} + frac{(-1)^{n+1}}{n!}right).$$



                  Notice that (again, since everything in sight converges absolutely) the second half of the $n$th term cancels with the first half of the $(n+1)$st term, so our sum telescopes to



                  $$frac{(-1)^2}{0!} = 1.$$






                  share|cite|improve this answer









                  $endgroup$


















                    2












                    $begingroup$

                    Note that, since all of the relevant sums converge absolutely



                    $$sumlimits_{i=1}^infty frac{(-1)^{n+1}(n+1)}{n!} = sumlimits_{i=1}^inftyleft(frac{(-1)^{n+1}}{(n-1)!} + frac{(-1)^{n+1}}{n!}right).$$



                    Notice that (again, since everything in sight converges absolutely) the second half of the $n$th term cancels with the first half of the $(n+1)$st term, so our sum telescopes to



                    $$frac{(-1)^2}{0!} = 1.$$






                    share|cite|improve this answer









                    $endgroup$
















                      2












                      2








                      2





                      $begingroup$

                      Note that, since all of the relevant sums converge absolutely



                      $$sumlimits_{i=1}^infty frac{(-1)^{n+1}(n+1)}{n!} = sumlimits_{i=1}^inftyleft(frac{(-1)^{n+1}}{(n-1)!} + frac{(-1)^{n+1}}{n!}right).$$



                      Notice that (again, since everything in sight converges absolutely) the second half of the $n$th term cancels with the first half of the $(n+1)$st term, so our sum telescopes to



                      $$frac{(-1)^2}{0!} = 1.$$






                      share|cite|improve this answer









                      $endgroup$



                      Note that, since all of the relevant sums converge absolutely



                      $$sumlimits_{i=1}^infty frac{(-1)^{n+1}(n+1)}{n!} = sumlimits_{i=1}^inftyleft(frac{(-1)^{n+1}}{(n-1)!} + frac{(-1)^{n+1}}{n!}right).$$



                      Notice that (again, since everything in sight converges absolutely) the second half of the $n$th term cancels with the first half of the $(n+1)$st term, so our sum telescopes to



                      $$frac{(-1)^2}{0!} = 1.$$







                      share|cite|improve this answer












                      share|cite|improve this answer



                      share|cite|improve this answer










                      answered Jan 16 at 15:14









                      user3482749user3482749

                      4,3191119




                      4,3191119























                          0












                          $begingroup$

                          $
                          sum_{n=1}^{infty}frac{(-1)^{n+1}(n+1)}{n!}\
                          =sum_{n=0}^{infty}frac{(-1)^{n}(n+2)}{(n+1)!}\
                          =sum_{n=0}^{infty}frac{(-1)^{n}}{n!}+sum_{n=0}^{infty}frac{(-1)^{n}}{(n+1)!}\
                          =sum_{n=0}^{infty}frac{(-1)^{n}}{n!}+sum_{n=1}^{infty}frac{(-1)^{n+1}}{n!}\
                          =sum_{n=0}^{infty}frac{(-1)^{n}}{n!}-sum_{n=1}^{infty}frac{(-1)^n}{n!}\
                          =1
                          $






                          share|cite|improve this answer









                          $endgroup$


















                            0












                            $begingroup$

                            $
                            sum_{n=1}^{infty}frac{(-1)^{n+1}(n+1)}{n!}\
                            =sum_{n=0}^{infty}frac{(-1)^{n}(n+2)}{(n+1)!}\
                            =sum_{n=0}^{infty}frac{(-1)^{n}}{n!}+sum_{n=0}^{infty}frac{(-1)^{n}}{(n+1)!}\
                            =sum_{n=0}^{infty}frac{(-1)^{n}}{n!}+sum_{n=1}^{infty}frac{(-1)^{n+1}}{n!}\
                            =sum_{n=0}^{infty}frac{(-1)^{n}}{n!}-sum_{n=1}^{infty}frac{(-1)^n}{n!}\
                            =1
                            $






                            share|cite|improve this answer









                            $endgroup$
















                              0












                              0








                              0





                              $begingroup$

                              $
                              sum_{n=1}^{infty}frac{(-1)^{n+1}(n+1)}{n!}\
                              =sum_{n=0}^{infty}frac{(-1)^{n}(n+2)}{(n+1)!}\
                              =sum_{n=0}^{infty}frac{(-1)^{n}}{n!}+sum_{n=0}^{infty}frac{(-1)^{n}}{(n+1)!}\
                              =sum_{n=0}^{infty}frac{(-1)^{n}}{n!}+sum_{n=1}^{infty}frac{(-1)^{n+1}}{n!}\
                              =sum_{n=0}^{infty}frac{(-1)^{n}}{n!}-sum_{n=1}^{infty}frac{(-1)^n}{n!}\
                              =1
                              $






                              share|cite|improve this answer









                              $endgroup$



                              $
                              sum_{n=1}^{infty}frac{(-1)^{n+1}(n+1)}{n!}\
                              =sum_{n=0}^{infty}frac{(-1)^{n}(n+2)}{(n+1)!}\
                              =sum_{n=0}^{infty}frac{(-1)^{n}}{n!}+sum_{n=0}^{infty}frac{(-1)^{n}}{(n+1)!}\
                              =sum_{n=0}^{infty}frac{(-1)^{n}}{n!}+sum_{n=1}^{infty}frac{(-1)^{n+1}}{n!}\
                              =sum_{n=0}^{infty}frac{(-1)^{n}}{n!}-sum_{n=1}^{infty}frac{(-1)^n}{n!}\
                              =1
                              $







                              share|cite|improve this answer












                              share|cite|improve this answer



                              share|cite|improve this answer










                              answered Jan 16 at 15:30









                              Fei XuFei Xu

                              12




                              12























                                  0












                                  $begingroup$

                                  Starting with
                                  $$e^{-x} = sum_{n=0}^{infty} (-1)^n frac{1}{n!} x^n$$
                                  Multiply by $x$
                                  $$x e^{-x} = sum_{n=0}^{infty} (-1)^n frac{1}{n!} x^{n+1}$$
                                  Differentiate with respect to $x$
                                  $$e^{-x}-x e^{-x} = sum_{n=0}^{infty} (-1)^n frac{n+1}{n!} x^n = 1 - sum_{n=1}^{infty} (-1)^{n+1} frac{n+1}{n!} x^n$$
                                  Now set $x=1$, with result
                                  $$0 = 1 - sum_{n=1}^{infty} (-1)^{n+1} frac{n+1}{n!} x^n$$
                                  so the sum is $1$.






                                  share|cite|improve this answer









                                  $endgroup$


















                                    0












                                    $begingroup$

                                    Starting with
                                    $$e^{-x} = sum_{n=0}^{infty} (-1)^n frac{1}{n!} x^n$$
                                    Multiply by $x$
                                    $$x e^{-x} = sum_{n=0}^{infty} (-1)^n frac{1}{n!} x^{n+1}$$
                                    Differentiate with respect to $x$
                                    $$e^{-x}-x e^{-x} = sum_{n=0}^{infty} (-1)^n frac{n+1}{n!} x^n = 1 - sum_{n=1}^{infty} (-1)^{n+1} frac{n+1}{n!} x^n$$
                                    Now set $x=1$, with result
                                    $$0 = 1 - sum_{n=1}^{infty} (-1)^{n+1} frac{n+1}{n!} x^n$$
                                    so the sum is $1$.






                                    share|cite|improve this answer









                                    $endgroup$
















                                      0












                                      0








                                      0





                                      $begingroup$

                                      Starting with
                                      $$e^{-x} = sum_{n=0}^{infty} (-1)^n frac{1}{n!} x^n$$
                                      Multiply by $x$
                                      $$x e^{-x} = sum_{n=0}^{infty} (-1)^n frac{1}{n!} x^{n+1}$$
                                      Differentiate with respect to $x$
                                      $$e^{-x}-x e^{-x} = sum_{n=0}^{infty} (-1)^n frac{n+1}{n!} x^n = 1 - sum_{n=1}^{infty} (-1)^{n+1} frac{n+1}{n!} x^n$$
                                      Now set $x=1$, with result
                                      $$0 = 1 - sum_{n=1}^{infty} (-1)^{n+1} frac{n+1}{n!} x^n$$
                                      so the sum is $1$.






                                      share|cite|improve this answer









                                      $endgroup$



                                      Starting with
                                      $$e^{-x} = sum_{n=0}^{infty} (-1)^n frac{1}{n!} x^n$$
                                      Multiply by $x$
                                      $$x e^{-x} = sum_{n=0}^{infty} (-1)^n frac{1}{n!} x^{n+1}$$
                                      Differentiate with respect to $x$
                                      $$e^{-x}-x e^{-x} = sum_{n=0}^{infty} (-1)^n frac{n+1}{n!} x^n = 1 - sum_{n=1}^{infty} (-1)^{n+1} frac{n+1}{n!} x^n$$
                                      Now set $x=1$, with result
                                      $$0 = 1 - sum_{n=1}^{infty} (-1)^{n+1} frac{n+1}{n!} x^n$$
                                      so the sum is $1$.







                                      share|cite|improve this answer












                                      share|cite|improve this answer



                                      share|cite|improve this answer










                                      answered Jan 16 at 21:32









                                      awkwardawkward

                                      6,82511026




                                      6,82511026






























                                          draft saved

                                          draft discarded




















































                                          Thanks for contributing an answer to Mathematics Stack Exchange!


                                          • Please be sure to answer the question. Provide details and share your research!

                                          But avoid



                                          • Asking for help, clarification, or responding to other answers.

                                          • Making statements based on opinion; back them up with references or personal experience.


                                          Use MathJax to format equations. MathJax reference.


                                          To learn more, see our tips on writing great answers.




                                          draft saved


                                          draft discarded














                                          StackExchange.ready(
                                          function () {
                                          StackExchange.openid.initPostLogin('.new-post-login', 'https%3a%2f%2fmath.stackexchange.com%2fquestions%2f3075841%2ffinding-the-sum-of-a-specific-alternating-series%23new-answer', 'question_page');
                                          }
                                          );

                                          Post as a guest















                                          Required, but never shown





















































                                          Required, but never shown














                                          Required, but never shown












                                          Required, but never shown







                                          Required, but never shown

































                                          Required, but never shown














                                          Required, but never shown












                                          Required, but never shown







                                          Required, but never shown







                                          Popular posts from this blog

                                          Human spaceflight

                                          Can not write log (Is /dev/pts mounted?) - openpty in Ubuntu-on-Windows?

                                          File:DeusFollowingSea.jpg